IAS 2020 Mock Test 1 - UPSC Prelims GS Paper I

IAS Toppers recommend solving UPSC Mock Tests as an essential part of Civil Services Exam preparation. In the week prior to the Prelims Exam Date, candidates should revise thoroughly and take as many IAS mock tests as possible. They should know that the UPSC test series ahead of the exams can help align their preparation.

Salient Features of Free OnlineMock Test for UPSC Prelims 2022

The important highlights of the UPSC Online Test series are as follows:

  1. 100 important model questions with solutions
  2. Free online test
  3. Section-wise sorted questions

In this article, aspirants will be provided two IAS Mock Tests. One, they can sit online right now and the other one which they can download and use for offline test purposes.

Solve IAS Mock Tests and align your UPSC preparation for the upcoming civil services exam. Complement your preparation with the links given below:

IAS Free Mock Test 1 – UPSC Prelims

Aspirants can download the GS-I Mock Test for UPSC Prelims from the link provided below. This test should be taken up by the candidates offline in an examination kind of environment while they can set a timer for 2 hours. This offline test can be used by the candidate as per his/her comfort as there are candidates who sometimes prefer to write a test for revision purposes.

UPSC GS-I Mock Test:-Download PDF Here

Given below is the first IAS mock test in our UPSC test series of free online mocks for IAS Exam Preparation which you can sit for right now. The details of the mock test are as follows:

CSE (Prelims) GS MOCK TEST

→ Question-1

Answer: a

Explanation:

The President is elected by members of the electoral college consisting of:

1. Elected members – both the Houses of Parliament;

2. elected members- legislative assemblies of the states; and

3. Elected members-legislative assemblies of the Union Territories of Delhi and Puducherry.

Statement 1 is correct: Following members do not participate in the election of the President-

  • Nominated members of both of Houses of Parliament,
  • the nominated members of the legislative assemblies of the States
  • The nominated members of the state legislative councils (in case of the bicameral legislature) and
  • The nominated members of the Legislative Assemblies(Delhi and Puducherry.

Statement 2 is correct:

Every elected member of state legislative assembly shall have as many votes as there are multiples of one thousand in the quotient obtained by dividing the population of the state by the total number of the elected members of the assembly.

Therefore, the value of the vote of the member of a legislative assembly is directly proportional to the population of the state and inversely proportional to the number of elected members of the assembly.

Statement 3 is incorrect:

The President’s election is held in accordance with the system of proportional representation by means of the single transferable vote and the voting is done by secret ballot system.

→ Question-2

Answer: a

Explanation:

Statement 1 is correct: The constitution has not mentioned the procedure related to the removal of the judge of the supreme court. The procedure is mentioned in the judges enquiry act(1968).

Statement 2 is incorrect: It is the President of India who can be impeached for the ‘violation of the constitution’. A judge of the supreme court can be removed on two grounds which are —proved misbehaviour or incapacity.

Statement 3 is incorrect: A person to be appointed as a judge of the Supreme Court should have the following qualifications:

1. He should be a citizen of India.

2. (a) He should have been a judge of a High Court (or high courts in succession) for five years; or (b) He should have been an advocate of a High Court (or High Courts in succession) for ten years; or (c) He should be a distinguished jurist in the opinion of the president.

From the above, it is clear that the Constitution has not prescribed a minimum age for appointment as a judge of the Supreme Court.

→ Question-3

Answer: a

Explanation:

Statement 1 and 2 are correct: The Organisation of Islamic Cooperation (OIC) is the second largest intergovernmental organization after the United Nations with a membership of 57 states spread over four continents such as Asia, Europe, Africa, South America. The Organization is the collective voice of the Muslim world. The OIC safeguardssand protects the interests of the Muslim so as to promote global harmony and peace.

Statement 3 is incorrect: India is neither a member nor an observer in this organisation.

→ Question-4

Answer: a

Explanation:

Statement 1, 2, 3 and 4 are correct:

  • Jizya, the tax that non-Muslims had to pay was abolished by Akbar in 1564.
  • He also abolished the pilgrim tax on bathing at holy places such as Prayag, Banaras etc. in 1563.
  • Akbar granted inam lands to all, irrespective religious faith and beliefs. This non-discriminatory policy favored his Hindu subjects. Sanads (deeds) of grant to various Hindu maths made by Akbar are still preserved.
  • In 1562, Akbar banned the forceful conversion of war prisoners. Akbar gave equal citizenship status to both Hindus and Muslims. His policy didn’t admit political differentiation on the basis of religion.

Statement 5 is incorrect: Cattle tax (Chari) and house tax (Ghari) were levied during the reign of Alauddin Khalji. It is not related to Akbar’s measures for the benefit of his Hindu subjects.

→ Question-5

Answer: c

Explanation: The fundamental principles of the Brahmo Samaj, founded by Raja Ram Mohan Roy in 1828 are:

1. There is only one God, who is the creator, infinite in power, wisdom, love, justice and holiness, omnipresent, eternal and full of bliss

2. The human soul is responsible to God for its doings. It is immortal and can progress infinitely.

3. Man’s happiness in this and the next world consists in worshipping God in spirit and in truth.

Statement 1 is incorrect: The long-term agenda of the Brahmo Samaj—to purify Hinduism and to preach monotheism—was based on the twin pillars of reason and the Vedas and Upanishads.

Statement 2 is correct: In 1881 the Brahmo Samaj at Barisal (Bengal) appointed the first woman Brahmo preacher (Manorama Mazumdar).

Statement 3 is incorrect: With the advent of railways the preachers of the Brahmo Samaj could travel afar, and the message of samaj soon spread to the rest of British India from Kolkata. Outside Bengal Presidency the prominent centres of Brahmo Samaj activity included Punjab, Sind and Madras Presidencies.

→ Question-6

Answer: a

Explanation: The Government of the day is headed by The Prime Minister

Statement 1 is correct: The Prime Minister is the leader of the Lower House. In this capacity, he enjoys the following powers:

1. The President is advised by the PM with regard to summoning and proroguing of the sessions of the Parliament.

2. He can recommend dissolution of the Lok Sabha to the President at any time.

3. He announces government policies on the floor of the House.

Statement 2 is correct:

The Prime Minister recommends persons who can be appointed as ministers by the President. The President can appoint only those persons as ministers who are recommended by the Prime Minister. He allocates and reshuffles various portfolios among the ministers.

Statement 3 is incorrect:

The Prime Minister is not the chairman of the Civil Services Board. The Cabinet Secretary is the chairman of the “Civil Services Board” (CSB). It decides on transfer and postings of bureaucrats.

→ Question-7

Answer: a

Explanation:

Statement 1 is incorrect: The term of office of the AG is not fixed by the Constitution. Further, the Constitution does not contain the procedure and grounds for his removal. He holds office during the pleasure of the president. This means that he may be removed by the president at any time. He may also quit his office by submitting his resignation to the president. Conventionally, he resigns when the government (council of ministers) resigns or is replaced, as he is appointed on its advice.

Statement 2 is correct: The Attorney General has the right of audience in all courts in the territory of India. Further, he has the right to speak and to take part in the proceedings of both the Houses of Parliament or their joint sitting and any committee of the Parliament of which he may be named a member, but without a right to vote. He enjoys all the privileges and immunities that are available to a member of Parliament.

Statement 3 is incorrect: The Attorney General is not a full-time counsel for the Government. He does not fall in the category of government servants. Further, he is not debarred from legal practice. There are certain limitations on the Attorney General in performing his duty-.

  1. He should not advise or hold a brief against the Government of India.
  2. He should not advise or hold a brief in cases in which he is called upon to advise or appear for the Government of India.
  3. He should not defend accused persons in criminal prosecutions without the permission of the Government of India.
  4. He should not accept appointment as a director in any company or corporation without the permission of the Government of India.

→ Question-8

Answer: b

Explanation: The Model Code of Conduct (MCC) is a set of rules put in force by the Election Commission of India to guarantee free and fair elections.

Statement 1 is incorrect: The Model Code of Conduct for guidance of candidates and political parties comes immediately into effect after the announcement of elections. The Commission normally announces the schedule of elections in a major press conference a few weeks before the formal process is set in motion. The formal process for the elections starts with the Notification or Notifications calling upon the electorate to elect Members of a House.

Statement 2 is correct: From the time elections are announced by the Election Commission, Ministers and other authorities shall not announce any financial grants in any form or promises thereof.

Statement 3 is incorrect: All parties and candidates shall avoid holding public meetings during the period of 48 hours ending with the hour fixed for the close of the poll.

→ Question-9

Ans:(b)

Explanation:

To respect the National Flag and the National Anthem is a Fundamental Duty and not National Song. Hence statement 1 is incorrect.

To safeguard public property, develop humanism and spirit of enquiry and to safeguard the forests and wildlife are Fundamental Duties. Hence option B is correct.

→ Question-10

Ans:(d)

Explanation:

STATEMENT 1 is incorrect as the state High Court can enforce both the state laws and central laws.

STATEMENT 2 is incorrect as The Parliament can establish a common high court for two or more states. For example, Maharashtra and goa or Punjab and Haryana have a common High Court.

→ Question-11

Ans:(b)

Explanation:

STATEMENT 1 is incorrect am Constitutional amendment is initiated in either House of Parliament and not state legislatures.

STATEMENT 2 is correct as the bill for constitutional amendment can be introduced either by a private member or a minister and without prior permission of the President.

STATEMENT 3 is incorrect as Each House must pass the bill separately. In case of a disagreement between the two Houses, there is no provision for holding a joint sitting

→ Question-12

Ans:(c) Explanation:

Statement 1 is correct

It provided for responsible governments at the provincial level (provincial autonomy). The 1937 elections to the provinces were based on it. lt also provided for bicameralism in some provinces.

Statement 2 is correct

It provided for the establishment of a federal court

→ Question-13

Ans:(c)

Explanation:

Centre’s property is exempted from all taxes imposed by a state or any authority within a state like municipalities, district boards, panchayats and so on. Hence statement 1 is correct.

Article 301 declares that trade, commerce and intercourse throughout the territory of India to be free. Thus It encourages the free flow of trade. Hence statement 2 is correct.

→ Question-14

Ans:(c)

Explanation:

1. Cooperative societies were meant primarily to safeguard the interests of farmers, producers, consumers, borrowers etc. from exploitation by businesses. The need for profit- making is balanced by the needs of the members of the cooperative as it is jointly owned.

2. They are not dependent on other entities. They manage their affairs on their own.

3. All members get an equal say in the affairs of the cooperatives.

4. Cooperatives are jointly owned.

Eg: Amul

Amul is an Indian cooperative dairy company.

→ Question-15

Answer: a

Explanation :

Statement 1 is correct: The Ecological Footprint measures the amount of biologically productive land and sea area an individual, a region, all of humanity, or a human activity that compete for biologically productive space. This includes producing renewable resources, accommodating urban infrastructure and roads, and breaking down or absorbing waste products, particularly carbon dioxide emissions from fossil fuel. Biologically productive land and sea includes cropland, forest and fishing grounds, and does not include deserts, glaciers and the open ocean.

Statement 2 is incorrect: According to the National Footprints Accounts (2014), India has an ecological footprint of 1.12 global hectares (gha) per person and a biocapacity of 0.45 gha per person which means it is a ‘biocapacity debtor’ or an ‘ecologically deficit country’ with there being a 148 per cent more demand than supply on its natural resources.

Additional Information

While both U.S.A. and India are consuming more than their ecological limits, an average U.S. citizen is creating an ecological burden many times that of the average Indian citizen. Looked at this way, it draws attention to uneven ecological burdens created by countries and emerges as a political rallying in international negotiations.

→ Question-16

Answer: d

Explanation

Statement 1 is correct: Pyrolysis is a method of solid waste management whereby solid wastes are chemically decomposed by heat without the presence of oxygen. This usually occurs under pressure and at temperatures of up to 430 degrees Celsius.

Statement 2 is incorrect: The root zone is a wastewater treatment system, and not a solid waste management method. It makes use of biological and physical-treatment processes to remove pollutants from wastewater.

Statement 3 is correct: Pulverisation is a method used to reduce the solid waste into dust or powder by pounding or grinding.

Statement 4 is correct: Incineration involves burning of solid wastes at high temperatures until the wastes are turned into ashes. Incinerators are made in such a way that they do not give off extreme amounts of heat when burning solid wastes.

→ Question-17

Answer: a

Explanation: The floating treatment wetland (FTW) is used for cleaning a lake by absorbing nutrients dissolved in the water, such as excess nitrates and oxygen with the help of plants, thereby reducing the content of these chemicals in the lake or a water body. An artificial island structure is created which floats on a water body. The island structure has plants above it which absorbs nutrients from the water body.

Statement 1 is correct: These wetlands are based on the principle of hydroponics whereby there is no need of soil to support the plant species. Hydroponics allows plants to grow only on sunlight and water, without the need of soil. There are small holes at the bottom of the island structure, which facilitates the flow of nutrients from the water to the plants [the biological uptake process].

Statement 2 is correct: These types of wetlands require building of artificial island structures in order to support the whole ecosystem. These are made with the support of gunny bags, bamboo and styrofoam. The FTW comprises four layers, where the 1st layer is made up of floatable bamboo forming the base of the island structure. The second layer comprises styrofoam cubicles. The third layer is composed of gunny bags, and Gravel forms the final layer, which helps to hold the nutrients.

Statement 3 is incorrect: Neknampur lake in Hyderabad, and not Loktak lake in Manipur is the largest FTW in India.

→ Question-18

Answer: c

Explanation: There are 4 biodiversity hotspots present in India. They are: The Himalayas, Indo- Burma, the Western Ghats & Sri Lanka, and Sundaland.

Statement 1 is correct: Includes the entire Indian Himalayan region (and that falling in Pakistan, Tibet, Nepal, Bhutan, China and Myanmar). This hotspot is home to the world’s highest mountains, diversity of vegetation, and important populations of numerous fauna species.

Statement 2 is correct: Western Ghats and Sri Lanka: Includes entire Western Ghats (and Sri Lanka).

Statement 3 is correct: Indo-Burma includes Myanmar, Thailand, Vietnam, Laos, Cambodia and southern China, and entire North-eastern India, except Assam and Andaman group of Islands. So, Andaman group of Islands is not a part of Indo-Burma hotspot.

Statement 4 is incorrect: Sundaland includes Indonesia, Malaysia, Singapore, Brunei, Philippines, and Nicobar group of Islands from India. However, it explicitly includes only Nicobar group of islands and not Andaman group of islands. There are two Island groups in Andaman and Nicobar chain, the Andaman Islands and Nicobar Islands, ( separated by Ten Degree Channel). Thus, Andaman group of islands is not a part of any of the four identified hotspots, whereas Nicobar group of islands is part of Sundaland hotspot.

→ Question-19

Answer: d

Explanation: Ozone is a gas composed of three atoms of oxygen (O3). Ozone is present in the Earth’s upper atmosphere as well as at ground level. It can be good or bad, depending on place of its occurence

Statement 1 is correct: Ground level ozone is formed due to the interaction of nitrogen oxides which are emitted from vehicles, power plants, industrial boilers, refineries, chemical plants, and other sources and volatile organic compounds.

Statement 2 is correct: Pollutants and volatile organic compounds chemically react in the presence of sunlight. Thus, sunlight is the prime requirement for such reactions resulting in the formation of ozone.

Statement 3 is incorrect: It does not filter out UV rays, but rather it is very harmful to the organisms and forms a major pollutant in the ecosystem. On the other hand, the upper atmospheric or stratospheric ozone protects us from harmful UV rays.

Statement 4 is correct: The ground level ozone also leads to the formation of photochemical smog which is a common occurrence in industrial cities around the world.

→ Question-20

Answer: d

Explanation: The KUSUM scheme consists of three components:

Component-A: 10,000 MW of Decentralized Ground Mounted Grid Connected Renewable Power Plants.

Component-B: Installation of 17.50 lakh standalone Solar Powered Agriculture Pumps.

Component-C: Solarisation of 10 Lakh Grid-connected Solar Powered Agriculture Pumps.

Statement 1 is correct: The objective of the scheme is providing financial and water security to farmers.

Statement 2 is correct: The Scheme will have substantial environmental impact in terms of savings of CO2 emissions. All three components of the Scheme combined together are likely to result in the saving of about 27 million tonnes of CO2 emissions per annum.

Statement 3 is correct: The scheme has direct employment potential. Besides increasing self-employment the proposal is likely to generate employment opportunities equivalent to 6.31 lakh job years for skilled and unskilled workers.

→ Question-21

Answer: a

Explanation:

Statement 1 is correct:

  • The Ministry of Labour and Employment issued a notification to categorize currency printing presses and mints as ‘public utility service’ under the Industrial Disputes Act, 1947.
  • Public utility services are those business undertakings engaged in supplying essential goods and/or services of daily necessity for the general public.
  • They have an obligation to supply the essential goods and services to everyone in the community without any discrimination at reasonable prices.

Statement 2 is incorrect:

  • As per the Industrial Disputes Act, 1947, no person employed in a public utility service shall go on strike in breach of contract without giving a notice to the employer.
  • The right to protest is a fundamental right under Article 19 of the Constitution of India. But the right to strike is a legal right ( under the Industrial Disputes Act, 1947) and not a fundamental right.

→ Question-22

Answer: d

Explanation:

Pair 1 is correctly matched: Minamata Disease is caused due to excessive amount of mercury present in the water. These can also be experienced with the intake of fish affected by mercury poisoning. They lead to numbness of limbs, deafness and other debilitating illnesses.

Pair 2 is correctly matched: Blue baby syndrome occurs due to excessive amount of nitrates present in the water. These nitrates then react with the haemoglobin of the blood leading to the formation of methaemoglobin which results in loss of oxygen carrying capacity of the blood.

Pair 3 is incorrectly matched: Black foot disease is caused by leaching of arsenic from soil & rocks to groundwater used for drinking purposes. Excessive fluoride in drinking water causes skeletal Fluorosis such as neuromuscular disorders, teeth deformity, hardening of bones, painful joints & outward bending of legs from knees, which is also known as Knock knee syndrome.

Pair 4 is correctly matched: Itai Itai or Ouch Ouch disease is caused due to excess amount of cadmium which pollutes the water. This disease causes a disabling pain in the joints and bones leading to locomotive issues.

→ Question-23

Answer: c

Explanation: Mangroves are a group of trees and shrubs that live in the coastal intertidal zone. They are one of the most productive and biologically diverse ecosystems on the planet. Mangroves can absorb up to four times more carbon dioxide by area than upland terrestrial forests. It benefits us with a wide variety of goods and services. These play a critical role in supporting human life with facilities like food, shelter and livelihoods. It also reduces loss of property and vulnerability of local communities.

Statement 1 is incorrect: Mangroves help filter out contaminants from the ecosystem including heavy metals and act as a sink thus keeping the coastal environment clean.

Statement 2 is correct: The leaves of these mangroves have salt secreting glands which push out excess amounts of salts from the system. These salts can often be found in the form of crystals deposited on the leaves. This helps them in coping with the saline environment.

Statement 3 is incorrect: These mangroves are a rich breeding ground for fishes and other faunal forms. The fish, crustaceans and a host of other species live between tree trunks in mangrove forests.

Statement 4 is incorrect: The roots of many of the mangrove plants themselves filter out the salts so that the water intaken by these plants is not saline. Thus, not all the plants are incapable of filtering out salt.

Statement 5 is correct: As they grow in an aquatic environment, these plants have lenticellated barks. Lenticel is a spongy area in the bark of a woody plant, serving as a pore to permit the exchange of gases between the stem and the atmosphere.

→ Question-24

Answer: c

Explanation: Bioremediation is the process of treating the polluted and contaminated soil or water with the help of microorganisms and plants.

Statement 1 is incorrect: This can be done both in situ (at the site) or ex situ (at a suitable place of choice).

Statement 2 is correct: Plants can be used to remove contaminants by the method of phytoremediation. By this they either stabilize the soil contaminants at one place, reducing its spread, or accumulating it above the soil.

Statement 3 is incorrect: By the method of bioremediation, a complete and total breakdown of pollutants and contaminants can be achieved leading to detoxification of the soil.

Statement 4 is correct: Bioattenuation is the process in which bioremediation occurs naturally without human intervention. Similarly, biostimulation is the process of stimulating biodegradation by creating the correct environmental conditions for microorganisms in the soil. In bioaugmentation, altered microorganisms are added to a contaminated site to accelerate detoxification and degradation.

→ Question-25

Answer: b

Explanation:

Statement 1 is incorrect:

It is a new social impact bond launched exclusively for women.

A social impact bond (SIB) is a contract with the legitimate authority, whereby it pays for better social outcomes in certain areas and passes on the part of the savings achieved to investors.

Statement 2 is correct:

Women livelihood bonds are unlisted and unsecured bonds. They offer fixed coupon rate of 3 per cent per year to an investors.

Unsecured bonds are also known as debentures, and are not backed by any collateral.

Unlisted securities are bonds or any other securities that are not traded on the stock exchange but through the over-the-counter (OTC) market.

Statement 3 is incorrect.:

SIDBI launched the bond along with World Bank and UN Women. It will have a tenure of five years.

The new bond will not only enable women self help groups to graduate from ‘group borrowing’ to ‘individual borrowing’ but will also allow them to shift from development assistance towards more market financed programmes.

→ Question-26

Answer: a

Explanation

Option (a) is correct: The EASE Reforms index is prepared jointly by the Indian Banking Association (IBA) and the Boston Consulting Group. It is a framework that was adopted by the finance ministry to strengthen public sector banks, and rank them on metrics such as responsible banking, financial inclusion, credit offtake and digitisation.

Option (b) is incorrect: There is no specific index for ranking states according to their service infrastructure. Their infrastructure overall is ranked under the National logistics index.

Option (c) is incorrect: There is no specific index for measuring the performance of power DISCOMs in terms of reach.

Option (d) is incorrect: Good governance index published by the Department of Administrative Reforms & Public Grievances rates ministries according to their performance in e-governance and other good governance initiatives.

→ Question-27

Answer: b

Explanation:

Statement 1 is correct: Inside the trunk of a tree, there are a number of growth rings called annual rings. A ring is added to the trunk every year. Thus, they are used to determine the age of a tree.

Statement 2 is correct: Pith is a tiny dark spot of living cells, present at the centre of the trunk of a tree. It carries essential nutrients to the tree.

Statement 3 is incorrect: Phloem is the inner layer of bark, which acts as a food supply line from the leaves to the rest of the tree. It carries sap (sugar and nutrients dissolved in water). Thus, it is a layer of bark. However, being an inner layer of the bark, it is made up of living cells. The outer layer of the bark is made up of dead cells.

→ Question-28

Answer: c

Explanation

Statement 1 is correct: The budget is granted to the government for one financial year, which starting from April of a current year to March of next year with due approval by Parliament. If the government cannot present a full budget before the financial year ends, due to some reasons, it needs Parliament’s authority for incurring expenditure in the new fiscal year till a full Budget is presented.

Statement 2 is incorrect: An Interim Budget is not similar to ‘Vote on Account’. ‘Vote on Account’ deals only with the expenditure side of the government’s budget, while an Interim Budget is a complete set of accounts, including both expenditure and receipts.

Statement 3 is correct: They are usually presented for a four-month period.

Extra Information

law does not debar the Union government from introducing tax changes, normally during an election year, however successive governments have avoided making such changes.

→ Question-29

Answer: a

Explanation:

Option a is correct: The Amendment Act of 2003 provided for the creation of a new type of protected area called a Conservation Reserve. It is an area owned by the State Government adjacent to National Parks and sanctuaries for protecting the landscape, seascape and habitat of fauna and flora. It is managed through a Conservation Reserve Management Committee. It generally connects the buffer zone around the national parks, wildlife sanctuaries and reserve forests with each other thus providing much needed continuity in the zones of conservation.

Option b is incorrect: Zoological parks are the protected areas where the animals are usually protected for awareness raising purposes and for recreational purposes for its visitors.

→ Question-30

Answer: c

Explanation: CPCB is a statutory organization which was formed in 1974 and is under the Ministry of Environment, Forests and Climate Change. This was formed under the provisions of the Water (Prevention and Control of Pollution) Act 1974 and was one of the country’s first organizations formed against environmental pollution. It is the apex body which monitors the various industrial and air and water pollution in our cities and comes out with advisories for the governments as well as for the municipalities involved.

Statement 1 is incorrect: It was established under the Water (Prevention and Control of Pollution) Act, 1974. This act also provides for the formation of state boards which shall be the bodies operating at the state level.

Statement 2 is correct: The Central Pollution Control Board is executing a nation-wide programme for ambient air quality monitoring known as National Air Quality Monitoring Programme (NAMP).

Statement 3 is incorrect: The CPCB can also impose financial penalties on the guilty parties including the municipal corporations who do not follow the rules regarding control of pollution. This body comes under the purview of the Ministry of Environment, Forests and Climate Change but the act ensures autonomy in its functions.

→ Question-31

Answer: B

Explanation:

At present, Chilika Lake is not part of Montreux Record. It was placed under Montreux Record due to siltation. It was removed from this record in 2002.

Loktak lake, Manipur was included on the Montreux Record in 1993 (signifying habitat degradation), as a result of ecological problems such as deforestation in the catchment area, infestation of water hyacinth and pollution.

Keoladeo national park, Rajasthan was placed on the Montreux Record in 1990 due to water shortage and unbalanced grazing regime around it.

→ Question-32

Answer: a

Explanation:

Option a is correct: National Mineral Policy, 2019 aims to bring more effective regulation to the mining sector as well as follow a more sustainable approach while addressing the issues of those affected by mining.

Under the new policy, the Right of First Refusal for Reconnaissance permit was introduced to encourage the private sector to take up exploration.

Earlier, if a firm finds evidence of minerals it has to inform the government under reconnaissance permit (RP). The government will then auction off the area.

But under the new policy, such firms will get the right of first refusal i.e. the same firm would be given the area for mining.

Thus, it encourages the private sector for mining.

→ Question-33

Answer: a

Explanation: AQI considers eight pollutants (PM10, PM 2.5, NO2, SO2, CO, O3, NH3 and Pb). Carbon dioxide(CO2) is not a component of the National Air Quality Index.

→ Question-34

Answer: d

Explanation: E waste management is one of the major issues the world is facing currently. With increased usage of electronic products, this is a problem on the rise. India in 2016 came out with an amendment to the E waste management rules. New provisions in the form of penalties and liabilities were included under the purview of E waste management.

Statement 1 is incorrect: As the management of e-wastes in general concerns both the electronic and electrical wastes. These wastes can be in the form of tube lights and CFLs and also small printed circuit boards and integrated circuits. These products if not managed and disposed properly, lead to land degradation as well as release of toxins in the environment.

Statement 2 is incorrect: Under these rules, there is an exemption provided to the micro industries from the Extended Producer Responsibility. Extended Producer Responsibility is a kind of policy approach whereby the producers are imposed with financial or physical responsibility to deal with the wastes generated in the course of usage of any product.

In this method, ploughing is done along the slope of the field.

→ Question-35

Answer: b

Explanation: Summer Ploughing is a type of tillage which helps to retain the soil moisture and help facilitate the nutrient content of the soil before the sowing of the kharif crops. It is mostly advocated in areas having less rainfall, so that whatever little rain occurs, it is utilized by the crops.

Statement 1 is incorrect: Under such agricultural practices, ploughing is done across the slope of the field rather than along the slope of the field. This not only ensures proper soil retention by preventing erosion, it also ensures maximum amount of water is retained in the process.

Statement 2 is correct: As ploughing exposes the layer lying at the bottom to the sun, thus baking it. Similarly, the heated layers then go to the bottom thus cooling them off. This leads to a better soil structure which is uniform.

Statement 3 is correct: As ploughing by this method leads to proper aeration being provided. This leads to a faster decomposition rate along with an improved environment as far as the growth of favourable micro organisms are concerned.

→ Question-36

Answer: a

Explanation: Dhrupad is a genre of Hindustani classical music.

Statement 1 is correct: Dhrupad was popular during the 14th century but flourished between 15th century onwards to around the 18th century. Man Singh Tomar, the Maharaja of Gwalior was primarily responsible for the popularisation of dhrupad.

Statement 2 is correct: Tansen was an famous dhrupad singer. Swami Haridasa was not only a famous dhrupad exponent but also follower of Bhakti cult. He was the guru of Tansen, one of the best known dhrupad singers and also one of the nine jewels of Emperor Akbar’s court.

Statement 3 is incorrect: There were four schools or vanis of singing the dhrupad.

The Gauhar vani that is-raga or unadorned melodic figures.

The Dagarvani that is- melodic curves and graces.

The Khandar vani that is-quick ornamentation of the notes. Nauhar vani was known for its broad musical leaps and jumps. Compared to the vanis of the dhrupads, we have gharanas, in the khyal. Various individuals or patrons( kings or noblemen) founded or established several several schools of singing. The oldest of them is the Gwalior gharana.

→ Question-37

Answer: b

Explanation: Indian music had evolved into two different subcategories :- Hindustani and Carnatic music.The two distinct styles, Hindustani and Carnatic became popular after the advent of the Muslims, particularly during the reign of the Mughal Emperors of Delhi. Both the systems of music received their nourishment from the same original source. Whereas the Indian music of the Northern part of India assimilated some features of the music of the Persian and Arabic musicians who adorned the courts of the Mughal rulers of Delhi, the music of the South continued to develop along its own original lines. But the fundamental aspects of both the systems of the North and South have been the same.

It is said, that South Indian Music, as known today, flourished in Deogiri the capital city of the Yadavas in the middle ages, and that after the invasion and plunder of the city by the Muslims, the entire cultural life of the city took shelter in the Carnatic Empire of Vijayanagar under the reign of Krishnadevaraya. Thereafter, the music of South India came to be known as Carnatic Music.

Statement 1 is incorrect: Gitam is the simplest type of composition. Taught to beginners of music, the gitam is very simple in construction, with an easy and melodious flow of music. The music of this form is a simple melodic extension of the raga in which it is composed. The Varnam is a beautiful creation of musical craftsmanship of a high order, combining in itself all the characteristic features of the raga in which it is composed.

Statement 2 is correct: Jatisvaram-has no sahitya or words. The piece is sung with solfa syllables only. Examples of solfa syllables a taka tari kita naka tatin gina tam

Statement 3 is correct: The Kirtanam had its birth about the latter half of the 14th century. It is valued for the devotional content of the sahitya. Clothed in simple music, the kirtanam abounds in Bhakti bhava. It is suited for congregational singing as well as individual presentation.

→ Question-38

Answer: d

Explanation:Many features of local cultures and traditions were combined with Islamic architectural practices during the medieval period. Thus, in the field of architecture, a mix of many structural techniques, stylised shapes, and surface decorations came about through constant interventions of acceptance, rejection or modification of architectural elements. These architectural entities or categories showcasing multiple styles are known as Indo-Saracenic or Indo-Islamic architecture.

Decorative forms in Indo-Saracenic architecture included designing on plaster through incision or stucco. They were influenced by Turkish, Persian and Indian traditions.

Statement 1 is correct: Tiles were also used to surface the walls and the domes. Popular colours were blue, turquoise, green and yellow.

Example:The tomb of Shah Rukn-i-Alam (Sufi saint) was built by the Tughluq ruler of Delhi, Ghiyas-ud-din (r. 1320-1325), between 1320 and 1324 AD. The exterior is the tomb is ornamented with the use of carved brick and wood as well as blue and white faience mosaic tiles with raised relief patterns.

Statement 2 is correct: Profuse use of jalis is a prominent feature of Indo-Saracenic architecture. Jalis were also used in Hindu temples. A fine example of Jali is seen at Tomb of Salim Chishti, Fatehpur Sikri shows Islamic geometric patterns developed in western asia. Jali is seen in Pattadakal Virupaksha temple and Chola architecture.

Statement 3 is correct: Carvings of bell and chain motifs showcase Indian influence in provincial styles of Indo – Islamic architecture.Amongst provincial styles, the architecture of Bengal and Jaunpur is regarded as distinct. Gujarat was said to have a markedly regional character for patrons borrowed elements from regional temple traditions such as toranas, lintels in mihrabs, carvings of bell and chain motifs, and carved panels depicting trees, for tombs, mosques and dargahs. The fifteenth century white marble dargah of Shaikh Ahmad Khattu of Sarkhej in Gujarat is a good example of provincial style and it heavily influenced the form and decoration of Mughal tombs.

Statement 4 is correct: Calligraphy was extensively used for decoration.Islamic verses were carved on the surfaces of Islamic monuments. Example:Taj Mahal

→ Question-39

Answer: d

Explanation: Sufism is a common term used for Islamic mysticism. The Sufis were very liberal in their religious outlook. They believed in the essential unity of all religions. They preached spirituality through music and doctrines that professed union with God. Sufism originated in Iran and found a congenial atmosphere in India under the Turkish rule. Their sense of piety, tolerance, sympathy, concept of equality and friendly attitude attracted many Hindus, mostly from lower classes, to Islam. Sufi saints such as Moinuddin Chisti, Nizamuddin Auliya, Fariduddin Ganj-e-Shakar were the pioneer sufïs who are still loved, respected and honoured in India

Pair 1 is correctly matched: Khanqah, the institutions (abode of Sufis) set up by the Sufis in northern India took Islam deeper into the countryside.

Pair 2 is incorrectly matched: Takia (resting place/retreat of a Muslim saint) also became the centres for the propagation of Islamic ideas. These were patronized both by the aristocracy and the common people. The Sufis emphasized respect for all human beings. Whirling meditation is a form of physically active meditation which originated among Sufis.

Pair 3 is correctly matched: Dargahs are shrines built over the graves of revered Muslim saints

Pair 4 is correctly matched: The Sufis were organised into religious orders or silsilahs. These silsilahs were named after their founders such as Chishti, Suhrawardi, Qadi. and Naqshbandis. According to Abul Fazl, the author of the Ain-i-Akbari, there were as many as fourteen silsilahs in India during the sixteenth century. Each order had its own khanqah, which served as a shelter for the Sufi saints and for destitutes, and later developed as a centre of learning.

→ Question-40

Answer: b

Explanation: Though illiterate Akbar patronised scholars and learned men. In his court there were nine such Navratna Mulla Do Pyaza, Hakin Human, Abdur Rahim Khan e Khanan, Abul Tayal, Tansen, Raja Todar Mal, Raja Man Singh, Faizi and Birbal.

Statement 1 is correct: Abdur Rahim Khan e Khanan was the son of Bairam Khan, Akbar’s trusted guardian and mentor, who was of Turkic ancestry.

Statement 2 is correct: Raja Todar Mal was instrumental in introducing a new system of revenue known as Zabt and a system of taxation called Dahsala. He was one of the ‘Navratnas’ of Akbar’s court.

Statement 3 is correct: Faizi was the brother of Abul Fazl. Akbar highly recognised the genius in him and appointed him tutor of his sons.Faizi is remembered for his works of poetry, numbering in the hundreds. One of his most well known books is Tabashir al-Subh, a collection of poems.

Statement 4 is incorrect: Abul Fazl Mamuri was a historian of the Mughal Empire during Aurangzeb’s reign and author of Tarikh-i-Aurangzeb, Tarikh-i-Abul Fazl Mamuri and co-author of Shahjahannama. It was Abu al-Fazal ibn Mubarak also known as Abu’l-Fazl, Abu’l Fadl and Abu’l-Fadl ‘Allami who was the Grand vizier of the Mughal emperor Akbar, and author of the Akbarnama, the official history of Akbar’s reign in three volumes, and a Persian translation of the Bible. He was also one of the Nine Jewels of Akbar’s royal court.

→ Question-41

Answer: a

Explanation :

Sher Shah organized a brilliant administrative system. The central government consisted of several departments. The king was assisted by four important ministers:

1. Diwan –i- Wizarat – also called as Wazir – in charge of Revenue and Finance.

2. Diwan-i-Ariz – in charge of the Army.

3. Diwan-i-Rasalat- Foreign Minister.

4. Diwan-i-Insha- Minister for Communications.

Statement 1 is correct: Foreign Minister was called Diwan-i-Risalat during Sher Shah’s administration.

Statement 2 is correct: The Revenue and Finance minister was called Diwan -i- Wizarat.

Statement 3 is incorrect: The minister in charge of the Army was called Diwan-i-Ariz. The Minister for Communications was called Diwan-i-Insha.

Statement 4 is incorrect: The Minister for Communications was called Diwan-i-Insha. The minister in charge of the Army was called Diwan-i-Ariz.

→ Question-42

Answer: a

Explanation

Statement 1 is incorrect: The founder of the Bahmani kingdom was Alauddin Bahman Shah (Hasan Gangu). The Kingdom was founded by him in the year 1347. And not by Firoz Shah Statement 2 is incorrect: Mahamud Gawan carried out many reforms in the Bahamani kingdom. He divided the kingdom into eight provinces called Tarafs. Each tarafs was governed by a tarafdar. In every province, a tract of land was set apart for the expenses of the sultan. Such land was called khalisa.

Statement 3 is correct: Ahmad Wali Shah shifted the capital from Gulbarga to Bidar. Earlier the capital of Bahamani Kingdom was Gulbarga.

→ Question-43

Answer: d

Explanation: The word Kathak has been derived from the word Katha which means a story. Kathakars or story-tellers, are people who narrate stories largely based on episodes from the epics, myths and legends. It probably started as an oral tradition. Mime and gestures were perhaps added later on to make the recitation more effective. Thus evolved a simple form of expressional dance, providing the origins of what later developed into Kathak as we see it today.

Statement 1 is correct: With the coming of the Mughals, this dance form received a new impetus. A transition from the temple courtyard to the palace durbar took place which necessitated changes in presentation. In both Hindu and Muslim courts, Kathak became highly stylised and came to be regarded as a sophisticated form of entertainment.Today, Kathak has emerged as a distinct dance form. Being the only classical dance of India having links with Muslim culture, it represents a unique synthesis of Hindu and Muslim genius in art. Further, Kathak is the only form of classical dance wedded to Hindustani or the North Indian music. Both of them have had a parallel growth, each feeding and sustaining the other.

Statement 2 is correct: The Kathak dancer is accompanied by an equally dextrous percussionist – the tabla player – whose fingers have to match the speed of the dancer’s feet and the sound of the ghungroos. The dancer or the tabla player recite a series of bols which are given physical interpretation by dance and musical accompaniment.

Statement 3 is correct: The nineteenth century saw the golden age of Kathak under the patronage of Wajid Ali Shah, the last Nawab of Oudh. He established the Lucknow gharana with its strong accent on bhava, the expression of moods and emotions.Another noteworthy patron of Kathak was Raja Chakradhar of Raigadh in Madhya Pradesh. He encouraged the gharanas – Lucknow as well as Jaipur.

→ Question-44

Answer: b

Explanation: The middle ages witnessed a series of battles which not only decided the fate of the kingdoms but in some cases changed the very course of history. These battles were not bound by any spatial or temporal limitations but were rather spread to the whole of the country. Able generalship and military advances in tactics made the warfare more than sheer numbers. While some of them led to the seeding of a new dynasty or clan in the society, some of them led to a virtual wipeout from the map of the country.

Statement 1 is correct: Battle of Tarain was fought between Prithviraj III of Chauhan Dynasty and Muhammad of Ghazni. In the second battle fought in 1192, Prithviraj lost the battle and the chauhan power in Delhi came to an end. They were then limited to a small principality near Ajmer (Ajayameru).

Statement 2 is incorrect: Battle of Chandawar was fought between Jayachandra and Muhammad Ghori in the year 1194. Defeat in this crucial battle paved the way for Muhammad to increase his conquest of the Ganga Valley and a gradual spread of Turkish rule in the country.

The battle between Babur and Medini Rai was fought at Chanderi in 1528 and helped babur consolidate his position in north India.

Statement 3 is correct: The Battle of Kanauj was fought between Sher Shah and Humayun in the year 1540. It was here that Humayun was met with a decisive defeat which left him as a king without a kingdom and he had to seek shelter abroad. Though accompanied by his younger brothers, able generalship helped Sher Shah carry the day and establish the reign of Sur dynasty over Agra.

Statement 4 is incorrect: The battle of Bannihatti or Talikota was fought in 1565 and marked an end to the once almighty Vijayanagar Empire. In the battle, a unified army of five Deccan Sultanates(Ali adil shah and four others) defeated the Vijayanagara Army and Vijayanagara General Ramaraju was killed in the battlefield.

→ Question-45

Answer: a

Explanation: An ecological pyramid is a diagrammatic representation of the relationship between different organisms in an ecosystem. These are referred to as pyramids because of their shape. A pyramid is a polyhedron for which the base is a polygon and all lateral faces are triangles. In the pyramid of numbers, the number of organisms in each trophic level is counted and presented.

Statement 1 is incorrect: They do not take the organism’s size into account so may not always resemble a pyramid. For example- The pyramid in the grassland ecosystem will be upright resembling a pyramid, and in case of tree ecosystem, it will be in the shape of inverted pyramid. However, in the case of a single tree it will not resemble pyramid shape, but it turns into spindle shape (having a circular cross section and tapering towards each end). The first figure in the image below is upright pyramid, the second image is spindle shaped, and the third image is inverted pyramid.

Statement 2 is correct: The number of individuals may decrease from lower trophic level to higher trophic level in the pyramid of numbers for grassland ecosystem, where producers are more than consumers. The number of individuals may also increase from lower trophic level to higher trophic level in a single tree ecosystem or the ecosystem having parasites.

Statement 3 is correct: The Pyramid of numbers in a single tree system is spindle shaped, where the lower trophic level has a single tree, then the number of caterpillars (primary consumers) is high, then again the number of blue tits (secondary consumer) decreases. See the second figure in the image given above, it resembles a spindle shape.

→ Question-46

Answer: a

Explanation: The emergence of Kannauj emerged as a centre of political power from the reign of Harsha onwards.

Statement 1 is correct: Pataliputra owed its power and importance to trade and commerce, and the widespread use of money. Tools were collected from trade who came from all sides . money became scarce after the decline of trade , Hence, soldiers and officers were paid through land grants, the city lost its importance. Power shifted to places having strategic importance and to military campaigns called skandhavaras, which dominated long stretches of land. Kannauj was a place of strategic importance. It was Situated in Farrukhabad (UP), and became politically prominent from the second half of the sixth century on.

Statement 2 is correct: It was difficult to fortify places along the plains but kannauj was situated on the elevated area which was easily fortifiable, whereas patliputra was situated in a plain area. Thus,because it was located right at the centre of the doab, Kannauj was well fortified in the seventh century, which helped in exercising control over the eastern and western wings of the doab, soldiers could be moved by both land and water routes.

Statement 3 is incorrect: The Sena Empire, a Hindu dynasty, ruled Bengal from the 11th to 12th century. Thus, attack by them on patliputra was not the reason for the shift of power from Patliputra to kannauj in post Gupta times.

→ Question-47

Answer: c

Explanation:

Statement 1 is correct: Arms Trade Treaty is the first legally binding international agreement to regulate the global trade in conventional arms by establishing common international standards for member countries. It has 102 state parties and was endorsed by the UN General Assembly and entered into force in December 2014.

Statement 2 is incorrect: Like Russia, China and Saudi Arabia, India has neither signed nor ratified the treaty. It was in the news because the USA is also planning to withdraw from the International Arms Trade Treaty, because the President views it as a threat to America’s second amendment right to bear arms.

Statement 3 is correct: It regulates ammunition or munitions fired, launched, or delivered by the conventional arms covered under the treaty. Nations also need to ensure the weapons they export do not end up being used for genocide, crimes against humanity or terrorist acts. If they do find out the arms will be used for any of these, they need to stop the transfer.

→ Question-48

Answer: d

Explanation

Statement 1 is correct: The Chola capital, Uraiyur was noted for its cotton trade. In ancient times, the Tamils traded with the Greek or Hellenistic kingdom of Egypt and Arabia, on the one hand, and with the Malay archipelago and China, on the other.

Statement 2 and 3 are correct: In the early centuries of the Christian era, the Chera state was as important as the states of the Cholas and Pandyas, and owed its position to trade with the Romans. Two regiments were set up by Romans at Muziris to protect their interest in the region.

Statement 4 is incorrect: Kadaisiyars were the members of the lowest class. Though their situation was a bit better than the earlier times, it was little different than slaves. Pariyars on the other hand were the agricultural labourers who also worked in animal skins and used them as mats.

Additional Information

Eventually many elements of Tamil culture spread to the north, and in the brahmanical texts, the Kaveri came to be regarded as one of the holy rivers of India.

→ Question-49

Answer: a

Explanation:

Statement 1 is correct: According to the early Samkhya philosophy, the presence of divine agency is not necessary for creation of the world. The world owes its creation and evolution more to Nature or prakriti than to God.

Statement 2 is correct: Nyaya, or the school of analysis, was developed as a system of logic. According to it, salvation can be attained through the acquisition of knowledge.

Statement 3 is incorrect: Mimamsa literally means the art of reasoning and interpretation. However, reasoning was used to provide justifications for various Vedic rituals, and the attainment of salvation was made dependent on their performance.

Additional Information: Salvation or moksha is the central subject of the texts on darshana or philosophy

→ Question-50

Answer: c

Explanation:

UNESCO Global Geoparks are single, unified geographical areas where sites and landscapes of international geological significance are managed with a holistic concept of protection, education and sustainable development.

Two sites in India have been idenfied by Geological Survey of India for Global Geoparks status- Lonar Lake (Maharashtra) and St. Mary’s Island and Malpe beach(Karnataka)

St. Mary’s Island, was declared a geo-heritage site in 1975 and is around 88-million-year-old a time when Greater India broke away from Madagascar.

Lonar crater became a geo-heritage site in 1979. It is relatively young , (50,000 years old).It is distinguished by a near-perfect, circular ejecta blanket, which refers to earth thrown up during the collision, around it.

→ Question-51

Answer. c

Explanation:

Option c is correct: State of World Population report(2019) was recently released by United Nations Population Fund, it was titled ‘Unfinished Business: the pursuit of rights and choices for all’. The year 2019 is the 50th anniversary of foundation of UNFPA’s .

The report depicts on the state of sexual and reproductive health in current scenario.

India accounts for over one-sixth of the world’s population in 2019

While India’s life expectancy at birth is lower than the world’s (69 years to 72), it scores higher than the world average in case 9f access to healthcare during childbirth, and also has a much lower adolescent birth rate.

→ Question-52

Answer. c

Explanation: 20th Livestock Census was released by Department of Animal Husbandry & Dairying under the Ministry of Fisheries, Animal Husbandry and Dairying . At present , India has the highest livestock population in the world at 125.5 crore.

Statement 1 is correct.

livestock data were collected online for the first time . The 20th livestock census is a unique attempt because for the first time such a major initiative has been taken for digitisation of household level data through online transmission from the field.

Statement 2 is incorrect.

The population of the total Exotic/Crossbred Cattle had shown an increase of 26.9 % in 2019 (compared to the previous census)

total Indigenous/ Non-descript cattle population seas a decline over the previous census.

Statement 3 is correct.

This 20th census is a breed-wise Livestock Census. The Goat population in the country in 2019 is 148.88 million showing an increase of 10.1% over the previous census.

The total Pigs in the country is 9.06 Million in the current Census, declined by 12.03% over the previous Census.

→ Question-53

Answer: c

Explanation:

Statement 1 is correct: 3D bioprinted human skin models have certain anatomically relevant structural, mechanical and biochemical features similar to native human skin.

Statement 2 is correct: The bioprinted skin model will have wide applications in testing cosmetics and can also reduce or have probability to replace testing on animals. The bioprinted skin is already in use by ITC Ltd for experiments.

BYJU’S monthly compilation May 2019

→ Question-54

Answer: b

Explanation

Statement 1 is incorrect

Kalinga is in modern day Orissa. It was bound by Mahanadi and Godaveri

Statement 2 is correct

A major part of Tamil Nadu was situated between the Krishna to the north and the Kaveri to the south.

Statement 3 is correct

The coastal area in the extreme south-west of the peninsula was covered by the modern state of Kerala. The sea coast along the western part of the peninsula is called the Malabar coast. Although the coast came to have several ports and small kingdoms, communications between the coast and the adjoining areas of Maharashtra, Karnataka, and Kerala were rendered difficult by the Western Ghats with difficult passes to cross.Territory of the Chera polity of the early historical period consisted of the present day central Kerala and western Tamil Nadu.

Extra Information

Each of the areas bounded by rivers, in some cases by mountains, and sometimes

with deltas and plateaus, constituted a political and administrative unit in which

different ruling dynasties rose and fell. On account of difficult communications

in a vast country and the defensibility of the natural frontiers, it was not easy for

the ruling class of one region to establish its control over all the other regions.

→ Question-55

Ans. D

Explanation:

Statement 1 is correct.

  • Hot pursuit refers to the urgent and direct pursuit of a criminal suspect by law enforcement officers under international rules of engagement for military forces
  • There is no international law governing ‘hot pursuit’ over land.

Statement 2 is correct.

  • United Nations Convention on the Law of the Sea, Article 111 of the treaty grants a coastal state the right to pursue and arrest ships escaping to international waters if pursuers are competent authorities of the state and have good reason to believe that the pursued ship has violated the state’s laws.

Statement 3 is correct.

  • The right of hot pursuit ceases as soon as the ship pursued enters the territorial sea of a foreign state.

Hot Pursuit

  • Hot pursuit owes its origin to the law of the seas against vessels that are involved in piracy or smuggling. The coastal country would take action in spite of the principle of freedom of the high seas — the rights of vessels of all nations to navigate freely on the high seas

Exceptions to Hot Pursuit

  • The right of hot pursuit ceases as soon as the ship pursued enters the territorial sea of a foreign state.
  • Where a coastal state, stopping or arresting a foreign ship outside the territorial sea on the basis of its right of hot pursuit, fails to justify the exercise, it shall be liable to compensate the ship for any loss or damage caused to it due to the exercise of this right.

Adaptation to land

  • There is no international law governing ‘hot pursuit’ over land. Over the years, this doctrine has been expanded on land, to justify the breaches of territorial sovereignty of foreign states as part of the ongoing pursuit of offenders.

→ Question-56

Answer: d

Statement 1 is incorrect: They were inhabited by hunters and gatherers.

Statement 2 is incorrect: The largest number of paintings of Bhimbetka belongs to the Mesolithic period. In this period there are multiple themes and smaller paintings with hunting scene predominates. Geometric scenes are a characteristic of paintings of the upper paleolithic period.

Statement 3 is incorrect: Bhimbetka paintings are of great variety, having themes like mundane aspects of life to sacred and royal images.

Additional information

These animal painting rituals were realistic in the context of hunting. animal paintings are conventional in Harappan context Animals continue to be depicted on the Harappan seals even though the people largely lived on the food they cultivated.

→ Question-57

Answer: b

Explanation:

Statement 1 is incorrect

The dominant framework of international intellectual property (IP) law — TRIPS (Trade Related Aspects of Intellectual property Rights) and the UPOV (Union for the Protection of Plant Varieties) — gives plant breeders exclusive rights over the varieties they develop, and mostly disregards customary rights of indigenous and farming communities to their genetic resources and associated knowledges.

Recognising the bias in international law, the Protection of Plant Varieties and Farmers’ Rights (PPV&FR) Act, 2001, entitles not just the breeder but also the farmer

Accommodating these twin purposes meant granting recognition of the proprietary claims of both the farmers and breeders, more accurately, of farmers as breeders (Section 2(c)). It gives the farmer the right to “save, use, sow, re-sow, exchange, share or sell” produce/seeds (S. 39 (1)(iv)). Importantly, the Indian farmer is permitted to even “brown bag seeds” — sell any variety of seed on the condition that they are sold in an unbranded form

Statement 2 is correct

Under the Indian law, the farmer has the right to “save, use, sow, re-sow, exchange, share or sell” produce/seeds.

→ Question-58

Ans – (C)

Explanation

Statement 1 is correct

The Vindhya mountains cut right across India from west to east and formed a boundary between north and south India. It acts as a language barrier as to its north, there is a predominantly hindi-language belt and to its south, predominantly dravidian languages dominate

Statement 2 is incorrect

Indo-Aryan languages form the highest spoken language group. The Dravidian languages, 153 in number, form the second major linguistic group of the country (24.47 per cent). After that, come sino-tebatan and Austro Asiatic languages respectively.

Statement 3 is correct

Among Dravidian languages, besides the four internationally known languages spread in many parts of the world, there are 26 Dravidian languages by the current count, of which 25 are spoken in India and one (Brahui) is spoken in Balochistan on the Pakistan-Afghanistan border.

Extra Information

Indian languages belong to four important groups: the Austro-Asiatic, Tibeto-Burman, Dravidian, and Indo-Aryan.

→ Question-59

Answer: b

Explanation

Statement 1 is incorrect

The deltaic portion of Bengal formed by the Brahmaputra and called Samatata, which was made to acknowledge the authority of Samudragupta in the fourth century, covered south-east Bengal.

Statement 2 is correct

In central Malwa and the adjoining parts of MP lay the state of Avanti. It was divided into two parts, the northern part with its capital at Ujjain, and the southern part at Mahishmati.

Statement 3 is correct

Atranjikhera was situated in the state of UttarPradesh.Antiquities belonging to the Kushan period have been unearthed from Atranjikhera in the district of Etah(UttarPradesh). The excavation revealed six periods of occupation of which period IV is assignable to early centuries of Christian era. The ceramic of this period consisted of a dominant red ware with medium fabric.

Statement 4 is incorrect

Indian culture also spread to Southeast Asia, but not through the medium of Buddhism. Except in the case of Burma it was mostly diffused through the brahmanical cults. The name Suvarnabhumi was given to Pegu and Moulmein in Burma, and merchants from Broach, Banaras, and Bhagalpur traded with Burma.

→ Question-60

Answer c

Explanation:

Statement 1 is correct.

  • It is an Android application which will be operational at the time of elections announcement.
  • Without having to rush to the office of the returning officer to lodge a complaint , citizens can immediately report on incidents of misconduct within minutes of having witnessed them.
  • The identity of the complainer will be kept confidential.

Statement 2 is incorrect.

  • The user will get 5 minutes to report an incident after having clicked a picture or a video.
  • To prevent any misuse, the app will not allow uploading of the pre-recorded or old images and videos.

Statement 3 is correct.

  • cVIGIL is an innovative mobile application for citizens to report Model Code of Conduct and Expenditure violations during the elections. cVIGIL provides time stamped evidentiary proof of model code of conduct / Expenditure Violation, having live photo/video with auto location data.

→ Question-61

Answer: a

Explanation

Statement 1 is correct: spread of tantricism dates back to sixth century A.D. and it is about this time that tantric texts, shrines and practices also appeared. It admitted both women and shudras into its ranks.

Statement 2 is correct: There was great attention on the use of magic rituals to cure day to day diseases and injuries as well as satisfy the material desires of devotees from physical possessions

Statement 3 is incorrect: Tantrism permeated Jainism, Buddhism, Shaivism, and Vaishnavism.it continued to hold ground throughout the medieval age(Right since 7 AD). It is called Vajrayana in Tibet Buddhism.

→ Question-62

Ans – C

‘Skai’ is an electric airplane which is powered by a hydrogen fuel cell.

Its rotorcraft are run using very light hydrogen fuel cells, thus It has range of 644 km and has freight carrying capacity of 454 kilograms i

it flies for up to 300 miles at speed 118 mph, with nlaround zero emissions.

It’s buiilt to carry up to five people or 1000 pounds of total payload in either piloted, ground-piloted or fully autonomous mode.

→ Question-63

Answer: a

Explanation :

Statement 1 is correct: English as the official language was introduced during the reign of Lord William Bentinck.

Under the rule of Lord William Bentinck following reforms were initiated.

Abolition of sati and other cruel rites (1829)

Suppression of thuggee (1830)

Charter Act of 1833

Resolution of 1835, and educational reforms

Introduction of English as the official language

Statement 2 is incorrect: Abolition of female infanticide was initiated under the rule of Lord Hardinge.

Statement 3 is incorrect: Abolition of human sacrifice was initiated under the rule of Lord Hardinge.

→ Question-64

Answer: c

Explanation:

Pair 1 is incorrectly matched: At Kanpur,Nana Saheb, the adopted son of the last Peshwa, Baji Rao II was refused the family title and, banished from Poona, was living near Kanpur. Nana Saheb expelled the English from Kanpur, proclaimed himself the Peshwa, acknowledged Bahadur Shah as the emperor of India and declared himself to be his governor. Sir Hugh Wheeler, commanding the station, surrendered on June 27, 1857.

Pair 2 is incorrectly matched: Begum Hazrat Mahal took over the reigns at Lucknow where the rebellion broke out on June 4, 1857 and popular sympathy was overwhelmingly in favour of the deposed Nawab. Her son, Birjis Qadir, was proclaimed the Nawab and a regular administration was organized with important offices shared equally by Muslims and Hindus. Henry Lawrence, the British resident, the European inhabitants and a few hundred loyal sepoys took shelter in the residency. The residency was besieged by the Indian rebels and Sir Henry was killed during the siege.

Pair 3 is correctly matched: The most outstanding leader of the revolt was Rani Laxmibai, who assumed the leadership of the sepoys at Jhansi. The Rani of Jhansi had died on the battlefield earlier in June 1858. Jhansi was recaptured through assault by Sir Hugh Rose.

Pair 4 is correctly matched: At Benaras a rebellion had been organized which was mercilessly suppressed, by Colonel Neil, who put to death all suspected rebels and even disorderly sepoys. By the end of 1859, British authority over India was fully re- established.

Additional Information

The British Resistance

Delhi — John Nicholson, Lieutenant Willoughby, Lieutenant Hudson

Kanpur- Sir Hugh Wheeler, Sir Colin Campbell

Lucknow- Henry Lawrence, Brigadier Inglis, Henry Havelock, James Outram, Sir Colin Campbell

Jhansi- Sir Hugh Rose

Benaras – Colonel James Neill

→ Question-65

Answer: (b)

Explanation

There was marked economic digression towards the end of the Mughal rule. The financial position of the state deteriorated rapidly towards the end of the Mughal rule. It was mainly because zamindars and rebellious elements refused to pay land revenue.

Statement 1 is incorrect:

Land revenue was collected through revenue farming (Ijara system) during this period. In this system, the state’s share was fixed and the intermediary/zamindar was allowed to extort at will from the peasants. The system was inimical to the peasants but strengthened the finances of the zamindars. It led to peasant rebellions and rural unrest. This also contributed to the increase in political power of the zamindars which was detrimental to the power of the central authority over the zamindars. This in turn led to the fragmentation of the polity. It adversely affected trade as each local magnate started taxing traders separately. Thus, the ijara system was in no way good for the economy. Its discontinuation could have had a positive effect on the economy. But it was not abolished.

Statement 2 is correct:

Corrupt officials misappropriated state revenue during this period. This had a considerable impact on the state exchequer.

Statement 3 is incorrect:

Even though they took part in trade, they did not place restrictions on trade by monopolising it.If anything, they only provided impetus to trade during the medieval period.

The Portuguese government tried to monopolise Indian ocean trade by exacting fees from Indian and Arab traders and even from private Portuguese traders.

Note: Tariffs/Fees on trade can be detrimental to trade.This can in turn affect manufacturing and production.

→ Question-66

Answer :d

Explanation:

The Permanent Settlement (also Permanent Settlement of Bengal) was introduced by Lord Cornwallis in 1793. It was an agreement between the British East India Company and the Landlords of Bengal to settle the Land Revenue .The Permanent Settlement was one of the most famous measures of Lord Cornwallis. It was agreed that the landlords would have perpetual and hereditary rights over the land, so long as they pay the fixed revenue to the British Government.

Objectives of permanent settlement system:

1.Placing revenue on a definite footing and making revenue collection sure and certain.

2.Forging an alliance between the zamindar class and the company.

3.Relieving the officials of revenue matters and engaging them to other spheres of administration.

4.Increase in agriculture productivity.

Additional information

Since any increase in land productivity was not subject to increase taxes, it was expected that Zamindars would make efforts to improve the conditions of the tenants. This system was that what company would levy from the zamindars was fixed as 10 percent out of 11 parts.the 11 th part was the remuneration of the zamindar. However , what the zamindar would levy from the peasants was left unsettled. This implies that more the value of 11 parts, more is the remuneration of zamindar’s 11th part.Thus,it was clear that if the productivity of land is improved, the company will have no right to demand anything in excess of what was already settled. But this belief of Cornwallis was belied later because unfortunately, the increased earnings were not spent on peasants. Instead, this led to increased luxuries and pleasures of the zamindars.

→ Question-67

Answer: d

Explanation:

Study in India scheme:

  • It is a joint initiative of the Ministry of Human Resource Development, Ministry of External Affairs and Ministry of Home Affairs to target foreign students by branding India as an attractive education destination.
  • The programme envisages participation of select reputed Indian institutes and universities by way of offering seats for international students at affordable rates.
  • The proposed fee waivers to meritorious foreign students in this policy will be decided by the Institute. The expenditure on the fee waiver will have to be borne by the Institute concerned. No additional cash flow from the Government is proposed for the same.
  • Ministry of Human Resource Development has approved ‘Study in India’ programme with, inter-alia, the following objectives:
    1. To improve the soft power of India with focus on the neighbouring countries and use it as a tool in diplomacy.
    2. To double India’s market share of global education exports from less than 1 percent to 2 percent.
    3. Increase in contribution of international students in the form of direct spends, indirect spends, spillover effects.
    4. Increase in global ranking of India as an educational destination.
    5. To reduce the export – Import imbalance in the number of International students.

→ Question-68

Answer: b

Explanation:

Statement 1 is correct:

The government has launched a web portal, Central Equipment Identity Register (CEIR), to facilitate blocking and tracing of stolen/lost mobile phones.

It aims to curtail the counterfeit mobile phone market, discourage mobile phone theft and protect consumer interest.

Statement 2 is incorrect:

The CEIR has been undertaken by the Centre for Development of Telematics under the Department of Telecom (DoT).

Statement 3 is incorrect:

The Zonal Integrated Police Network (ZIPNET) is not integrated with the CEIR.

CEIR acts as a central system for all network Operators to share black listed mobile devices so that devices blacklisted in one network will not work on other networks even if the Subscriber Identity Module (SIM) card in the device is changed.

→ Question-69

Answer: a

Explanation:

Statement 1 is correct:

  • Previously the central government could designate an organisation as a terrorist organisation, if it, prepared or participated or was otherwise involved in terrorism.
  • Now the government is empowered to designate individuals as terrorists on the same grounds.

Statement 2 is correct:

  • Earlier the law required that NIA take prior permission from the respective state DGP to attach the proceeds of terrorism. The Amendment gives powers to Director General, NIA to attach properties acquired from proceeds of terrorism.

Statement 3 is incorrect:

  • The Home Minister said in the Parliament the law does not take away powers of the state police. Currently, the law requires that NIA take prior permission from the respective state DGP to attach the proceeds of terrorism. This delays the process as often such properties are in different states. So to reduce delays NIA is adequately empowered.
  • The Minister also said that the NIA’s conviction rate is 91%, which is exceptional by global standards.

→ Question-70

Answer: a

Explanation:

Statement 1 is incorrect: In Permanent Settlement, there was a provision for zamindar to collect the revenue. In the permanent settlement, the company recognized them as owners of soil. They were given permanent hereditary rights to collect revenue. So in permanent settlement, there were provisions for intermediary in the form of zamindar.

Statement 2 is incorrect: In the Mahalwari system the ownership and occupancy right was reserved for individual peasants. Even cultivation was to be done individually. But for the payment of the land revenue, the peasants were jointly responsible. Usually the village as a whole would be designated a Mahal and it paid the revenue via its headman called Lambardar. Thus, Lambardars worked as a link between the individual tillers and the government. So Lambardars act as intermediary between government and peasants.

Statement 3 is correct: In Ryotwari System the ownership rights were handed over to the peasants. British Government collected taxes directly from the peasants. So there were no provisions for intermediary in this land revenue system. Ryotwari System was introduced by Thomas Munro in 1820. Major areas of introduction include Madras, Bombay, parts of Assam and Coorgh provinces of British India.

→ Question-71

Answer: a

Explanation :

Jyotiba Phule organised a powerful movement against upper caste domination and brahminical supremacy. His works, Sarvajanik Satyadharma and Gulamgiri became sources of inspiration for the common masses. He was a pioneer of the widow remarriage movement in Maharashtra. Mahadeo Govind Ranade also worked for widow remarriage but, the book Sarvajanik Satyadharma was not written by him.

Phule, a firm believer in gender equality, was a pioneer in women’s education; he with the help of his wife, Savitribai, opened a girls’ school at Poona. He also opened a home for widows in 1854. Phule was awarded the title ‘Mahatma’ for his social reform work.

→ Question-72

Answer: b

Explanation:

  • Recently, Centre granted “in-principle” clearance for uranium exploration in Amrabad Tiger Reserve in Telangana.
  • India’s second-largest tiger reserve Amrabad is next only to its sibling, the original Nagarjunasagar Srisailam Tiger Reserve. Together they form what is probably India’s largest protected dry forest.
  • It lies in the Nallamala hills, a landscape that is recovering after over two centuries of degradation by the British and the Nizam of Hyderabad. Environmentalists argue the mining will destroy the landscape rich in biodiversity and one that supports a hunter-gatherer Scheduled Tribe.

→ Question-73

Answer: a

Explanation :

Option (a) is correct :

The Sarda Act was an important legislation for social reforms in British India. It was related to control the child marriage. The Sarda Act (1930) pushed up the marriage age to 18 and 14 for boys and girls. It is popularly known as the Sarda Act, after its sponsor Harbilas Sarda. The Act lacked implementation from the British Indian government, largely due to the fear of British authorities losing support from their loyal Hindu and Muslim communalist groups.

Option (b) is incorrect :

The Female Infanticide Prevention Act, 1870, also Act VIII of 1870 was a legislative act passed in British India, to prevent murder of female infants. The Act was introduced during the reign of governor general Lord Hardinge I. The Act made it compulsory for parents to register the birth of all babies and provided for verification of female children for some years after birth, particularly in areas where the custom was resorted to in utmost secrecy.

Option (c) is incorrect :

Due to the efforts of Pandit Ishwar Chandra Vidyasagar (1820-91), the principal of Sanskrit College, Calcutta, that the Hindu Widows’ Remarriage Act, 1856, was passed. It legalised marriage of widows and declared issues from such marriages as legitimate. Vidyasagar

cited Vedic texts to prove that the Hindu religion sanctioned widow remarriage.

Option (d) is incorrect :

Due to the efforts of Raja Rammohan Roy, the government declared the practice of sati illegal and punishable by criminal courts as culpable homicide. The regulation of 1829 was applicable in the first instance to Bengal Presidency alone, but was extended in slightly modified forms to Madras and Bombay Presidencies in 1830.

→ Question-74

Answer: b

Explanation

Recently, the Lok Sabha passed Transgender Persons (Protection of Rights) Bill, 2019.

Statement 1 is correct:

  • The bill defines a transgender person as one whose gender does not match the gender assigned at birth.

Statement 2 is incorrect:

  • Bill does not provide for any reservations in the field of employment and education as directed by the Supreme Court in the National Legal Services Authority (NALSA) verdict.
  • The Supreme Court recognized transgenders as the third gender in a landmark NALSA ruling. “Recognition of transgenders as a third gender is not a social or medical issue but a human rights issue,” the court said.

Statement 3 is incorrect:

  • The bill contravenes the Supreme Court’s judgment in NALSA v Union of India (UOI), which guarantees the right to self-identification without the need for medical intervention. They cannot self-identify their gender themselves. They have to obtain a certificate of identity from the District Magistrate.
  • The Bill appears to continue to mandate sex reassignment surgery for transgender people.

→ Question-75

Answer : b

Explanation :

Statement 1 is incorrect: Narayan Malhar Joshi founded the Social Service League.

Statement 2 is correct: It was mainly operating in the region of Bombay.

Social Service league :

  • A follower of Gokhale, Narayan Malhar Joshi founded the Social Service League in Bombay with an aim to secure for the masses better and reasonable conditions of life and work.
  • They organised many schools, libraries, reading rooms, day nurseries and cooperative societies.
  • Their activities also included police court agents’ work, legal aid and advice to the poor and illiterate.

→ Question-76

Answer: a

Explanation:

  • Recently a team of astronomers from Royal Astronomical Society have defined a new class of celestial objects called ‘Ploonets’.
  • Ploonets are the orphaned moons that have escaped the bonds of their parent planet and start orbiting their stars instead.
  • They could help explain some bizarre exoplanetary features and can also provide details on planet formation processes.

→ Question-77

Answer: b

Explanation:

  • The Government of Nagaland has decided to set up a Register of Indigenous Inhabitants of Nagaland (RIIN). It will be the master list of all indigenous inhabitants of the state based on an extensive survey.
  • Once the RIIN is finalised, no fresh indigenous inhabitant certificates will be issued. The only exception is newborn babies of the indigenous inhabitants of Nagaland.
  • The entire exercise will be monitored by the Commissioner of Nagaland.

→ Question-78

Answer: c

Explanation:

The National Defense Authorization Act for fiscal year 2020, which contains the proposal to bring India on par with North Atlantic Treaty Organization (NATO) allies, was passed by the US Senate recently.

The Act also provides for increased US-India defence cooperation in the Indian Ocean in the areas of humanitarian assistance, counterterrorism, counter-piracy and maritime security.

The upgrade will also smoothen the passage of sales of high–end US military hardware to India.

The US has already recognized India as a “major defence partner” in 2016.

→ Question-79

Answer: d

Explanation:

The Union Budget has proposed to introduce 20 percent tax on buybacks by listed companies. Until now, such provisions were applicable only to unlisted companies.

Statement 1 is correct:

  • A buyback is essentially a scheme by which a company repurchases a certain amount of its outstanding shares.

Statement 2 is correct:

  • Many companies were avoiding dividend payouts because of the dividend distribution tax (DDT). So the companies were returning cash to shareholders through share buybacks.
  • The value of buybacks went up from ₹1,724 crore in 2015 to ₹39,246 crore in 2018. In this process, there is a drop in government revenue as it was not getting paid DDT.
  • The proposed tax on share buybacks is aimed at plugging this loophole.

Statement 3 is correct:

  • The tax is now applicable to both listed and unlisted companies.

→ Question-80

Answer: d

Explanation:

Statement 1 is correct: Angel tax is levied on the capital raised via the issue of shares by unlisted companies from an Indian investor if the share price of issued shares is seen in excess of the fair market value of the company. The excess realization is considered as income and it is taxed at rate of 30.9% on net investments.

Statement 2 is correct: The tax, under section 56(2)(viib), was introduced in 2012 to fight money laundering. The stated rationale was that bribes and commissions could be disguised as angel investments to escape taxes. But given the possibility of this section being used to harass genuine startups, it was rarely invoked.The angel tax could not be scrapped as money laundering is a major problem. There is a network of 200 shell companies and they have been under control since 2012, so it cannot be scrapped.

Statement 3 is correct: Angel tax is imposed only on investments made by a resident investor. But the angel tax is not applicable in case the investments are made by any non-resident or venture capital funds.

→ Question-81

Answer: d

Explanation:

Foriegn Direct Investments in India is allowed under three categories of 100% FDI through automatic route; 100% FDI through government route and 100% FDI through government + automatic route. The sensitivity of the sector under consideration determines the category of investments allowed to it. Some of the sectors eligible for 100% FDI through automatic route are as follows:

  1. Agriculture and Animal Husbandry
  2. Non Scheduled air transport services
  3. Airports
  4. Automobiles
  5. Broadcasting carriage services
  6. Coal and Lignite mining
  7. Chemicals manufacturing
  8. Food processing industries
  9. Renewable Energy
  10. Tourism and hospitality

In the case of digital media like FDI is allowed upto 26% and that too under the government approval route. Hence the except option 5, all other options are correct.

→ Question-82

Answer: a

Explanation: A structural adjustment is a set of economic reforms that a country must adhere to in order to secure a loan from the International Monetary Fund and/or the World Bank. Structural adjustments are often a set of economic policies, including reducing government spending, opening to free trade and so on.

Structural adjustment programs have demanded that borrowing countries introduce broadly free-market systems coupled with fiscal restraint—or occasionally outright austerity. Countries have been required to perform some combination of the following:

  • Devaluing their currencies to reduce balance of payments deficits.
  • Cutting public sector employment, subsidies, and other spending to reduce budget deficits.
  • Privatizing state-owned enterprises and deregulating state-controlled industries.
  • Easing regulations in order to attract investment by foreign businesses.
  • Closing tax loopholes and improving tax collection domestically.

→ Question-83

Answer: d

Explanation:

Option (a) is correct: Global Competitiveness Report-published by World Economic Forum (WEF). This monitors the performance of countries based on a set of 12 categories called ‘pillars of competitiveness’, namely institutions, infrastructure, macroeconomic environment, health and primary education, higher education and training, goods market efficiency, labour market efficiency, financial market development, technological readiness, market size, business application and innovation.

Option (b) is correct: Global IT Report-WEF along with INSEAD, and Cornell University publishes this report which examines the increasing proliferation of technology and its effects on advancing global prosperity.

Option (c) is correct: Global Risk Report-Published by WEF. It enlists the threats which the world will face in future ranging from geopolitical and geo-economic tensions to environmental degradation and disruptions of the Fourth Industrial Revolution.

Option (d) is incorrect: World Economic Outlook-Published by IMF. The World Economic Outlook (WEO) is a report by the International Monetary Fund that analyzes key parts of the IMF’s surveillance of economic developments and policies in its member countries. It also projects developments in the global financial markets and economic systems. The WEO is usually prepared twice a year and is used in meetings of the International Monetary and Financial Committee.

→ Question-84

Answer: a

Explanation-

Statement 1 is correct: The OECD is an intergovernmental economic organisation,founded to stimulate economic progress and world trade.Most OECD members are high-Income economies with very high Human Development Index(HDI)and are regarded as developed countries.It was founded in 1961 and headquartered in paris.

Statement 2 is correct: OECD publishes reports like-

  • Government at a Glance 2017 report
  • International Migration Outlook
  • OECD Better Life Index

Statement 3 is incorrect: India is neither a member nor an observer but a key economic partner.

→ Question-85

Answer:d

Explanation:

Statement 1 is incorrect: Under Article 279 (1), The President of India has the authority to constitute the GST council.

Statement 2 is incorrect: The GST council is a constitutional body. The GST council will take all important decisions regarding the GST. The GST Council decides the tax rate, tax exemption, the due date of forms, tax laws, and tax deadlines, keeping in mind special rates and provisions for some states. The predominant responsibility of the GST Council is to ensure to have one uniform tax rate for goods and services across the nation.

→ Question-86

Answer: a

Explanation:

Statement 1 is correct: The International Finance Corporation (IFC) is an international financing institution that is a part of World Bank Group and offers investment, advisory, and asset-management services to encourage private-sector development in developing and less developed countries. in the United States. Since inception it has expanded its membership to 184 countries and extended its developmental assistance to countries from Asia-Pacific, Africa,Latin America and other developed regions.

Statement 2 is correct: IFC can be directly approached by a company or entrepreneur seeking to establish a new venture or expand an existing enterprise. The investment proposal can be submitted to the IFC, which then analyses the proposal and grants clearance based on credibility of the business/enterprise.

Statement 3 is incorrect: The IFC has extended debt financing to many private companies for Liquefied Natural Gas projects, Natural Gas exploration etc.

→ Question-87

Answer : d

Explanation:

Statement 1 is incorrect: Special drawing rights are supplementary foreign exchange reserve assets defined and maintained by the International Monetary Fund (IMF). SDR is not a currency rather they are units of account for the IMF. The aim was to supplement foreign exchange reserve assets, namely gold and U.S. dollars.

Statement 2 is correct: SDRs are allocated by the IMF to countries, and cannot be held or used by private parties. They provide liquidity to the global economic system and supplement member countries’ official reserves .

Statement 3 is incorrect: The SDR is neither a currency nor a claim against IMF assets, but a potential claim against the freely usable currencies of IMF members.The member states can exchange them for freely usable currencies IMF member countries can borrow SDRs from its reserves at favorable interest rates, mostly to adjust their balance of payments to favorable positions.

→ Question-88

Answer: a

Explanation:

Statement 1 is correct: The Asian Infrastructure Investment Bank (AIIB) is a multilateral regional investment bank. Its aim is to support the building of infrastructure in the Asia-Pacific region. The bank presently has 76 members and 26 prospective members from around the world. Established on intergovernmental lines, The AIIB is a multilateral development organization for Asia.

Statement 2 is correct: India along with 56 members was the founding member of Asian infrastructure investment bank and holds a stake of 7.5% which is the second largest after China.

Statement 3 is incorrect: The Asian Infrastructure Investment Bank (AIIB) began to offer local-currency financing to private-sector clients from India, Indonesia, Thailand, Turkey and Russia, in 2019. In its articles of agreement it has been stated that the bank would mobilise and support both private and government led projects.

→ Question-89

Answer: a

Explanation:

Statement 1 is incorrect: The New Development Bank (BRICS Bank), is a multilateral development bank established by the BRICS states (Brazil, Russia, India, China and South Africa). It was set up to counterbalance the Western-led financial institutions like the World Bank and the IMF by providing funding for infrastructure and development projects in BRICS countries. Each nation will have an equal say in the bank’s management, regardless of GDP size and capital subscriptions unlike world bank.

Statement 2 is incorrect: One of the major objectives of banks is to support Economic development projects in Asian and African Countries. The bank aims to contribute to development plans established nationally through projects that are socially, environmentally and economically sustainable.

Statement 3 is incorrect: The NDB is viewed as a competitior to Western-led global-development institutions such as the International Monetary Fund (IMF) and the World Bank.

Statement 4 is correct: The New Development Bank has an initial subscribed capital of US$50 billion and an initial authorized capital of US$100 billion. The initial subscribed capital is equally distributed among the founding members. The payment of the amount initially subscribed by each founding member to the paid-in capital stock of the Bank will be made in dollars in seven installments. If a member wants to increase its share of capital, it needs the consent of remaining four members.

→ Question-90

Answer: a

Explanation:

Statement 1 is correct: Forex reserves or foreign exchange reserves (FX reserves) are assets that are held by a nation’s central bank or monetary authority. Forex is used to back the liabilities – like the native currency issued and also reserves deposited by financial institutions or the government with the central bank.

The forex helps in keeping the value of their currency at a fixed rate.

Statement 2 is incorrect: In a floating exchange rate system the currency price of a nation is set by the forex market based on supply and demand relative to other currencies. On the other hand, in fixed exchange rates, the rates are entirely or predominantly determined by the government. Countries with a floating exchange rate system use forex reserves to stabilize their currency with respect to the US Dollar.

Statement 3 is incorrect: Currency appreciation refers to an increase in value of the currency compared to other currencies whereas depreciation is a fall in its value, which can affect the trade deficit. The trade deficit may worsen if the local currency appreciates because appreciation makes the imports cheaper and exports less profitable. Appreciation is directly linked to demand. If the value of the currency appreciates (or goes up), demand for the currency also rises. On the other hand, if the value of currency depreciates, it loses value against the currency against which it is being traded. The appreciating rupee helps boost returns in dollar terms for foreign investors pumping money in Indian markets.

→ Question-91

Answer: b

Explanation:

Statement 1 is correct: ‘G20 Global Smart Cities Alliance on Technology Governance’ is a league of 15 of world’s leading city networks and technology governance organizations. Their agenda will be to promote responsible and ethical use of smart city technologies. India’s also joined the league which is a critical first step towards accelerating global best practices, fostering greater openness and public trust as well as mitigating risks regarding the collection of data in public spaces.

Statement 2 is incorrect: It was established in June 2019, in conjunction with the G20 Summit held in Osaka, Japan.

Statement 3 is correct: India has joined the league that will work towards advancing the responsible and ethical use of smart city technologies.

Statement 4 is correct: The partners represent more than 2, 00,000 cities and local governments, leading companies, start-ups, research institutions, and civil society organizations. The World Economic Forum serves as the secretariat.

→ Question-92

Answer: a

Explanation:

Statement 1 is correct: The Systemically Important Banks are perceived as banks that are ‘too big to fail’. This perception of TBTF creates an expectation of government support for these banks at the time of distress. Due to this perception, these banks may resort to risk taking and there could be lack of market discipline on behalf of these banks. Hence, there is a need for a stronger regulatory environment for the SIBs. In this regard, the Basel Committee on Banking Supervision (BCBS) came out with a framework in November, 2011 for identifying the Global Systemically Important Banks (G-SIBs).

Similarly, the RBI has been mandated to identify the Domestic Systemically Important Banks (D-SIBs) and lay down suitable regulatory requirements to prevent their failure. In order to identify the D-SIBs, the RBI takes into account only those banks whose size is equal to or more than 2% of GDP. Further, these banks are categorized as D-SIBs only when they fulfill the mentioned criteria: size, interconnected-ness, lack of readily available substitutes or financial institution infrastructure, and complexity.

Statement 2 is incorrect: Presently, the SBI bank, ICICI bank and HDFC bank have been identified as Domestic Systemically Important Banks (D-SIBs).

→ Question-93

Answer: b

Explanation:

  • The US has urged Bangladesh to postpone the relocation of Rohingyas to the newly built camp on uninhabited island of Bhashan Char.
  • The Bhashan Char has emerged less than two decades back from the sea and according to the environmentalists it is an ecologically fragile area which is prone to floods, erosion and cyclone.

→ Question-94

Answer: c

Explanation:

The first edition of the School Education Quality Index (SEQI) has been released by NITI Aayog.

Statement 1 is incorrect:

  • SEQI helps in evaluating the performance of States andUTs in the school education sector.

Statement 2 is correct:

  • It groups the states and UTs as Large States, Small States and Union Territories in order to facilitate like-to-like comparisons. Kerala occupied first rank among 20 large states in terms of quality of school education, followed by Rajasthan and Karnataka, while Uttar Pradesh was ranked at the bottom position during 2016-17.
  • Manipur topped the list of small states followed by Tripura and Goa .

Statement 3 is incorrect:

  • Among the seven UTs Chandigarh was ranked first followed by Dadra and Nagar Haveli and Delhi .

→ Question-95

Answer: a

Explanation:

Statement 1 is incorrect: It is a joint military exercise and not naval exercise between India and France. The joint military exercise of India and French troops was held between 31st October and 13th November 2019 in Rajasthan.

Statement 2 is incorrect: The exercise is a military exercise and hence cannot showcase its naval dominance over the entire extended area of the Indian Ocean region.  It’s a biennial exercise and is conducted alternately in India and France. The bilateral training exercise was conducted at Foreign Training Node at Mahajan Field Firing Ranges, Rajasthan.

Statement 3 is correct: The joint exercise will focus on Counter Terrorism operations in backdrop of semi-desert terrain under United Nations Mandate. The exercise aims at enhancing understanding, cooperation and interoperability between the two Armies.

→ Question-96

Answer: c

Explanation:

Statement 1 is correct: HAM brings certainty in the returns to the investments by the private players. For the investment made by the Private they will be paid a variable annuity amount after the completion of the project depending on the value of the assets created. This arrangement reduces financial risks and increases liquidity as there is availability of credit due to certainty in returns.

Statement 2 is incorrect: Under the Hybrid Annuity Model (HAM) the government will contribute 40% of the project cost in the first five years through annual payments. Remainig 60% of the project cost has to be raised by the private developer in the form of equity or loans. So the majority of the investments are not made by the Government.

Statement 3 is correct: After completion of the project the National Highways Authority of India (NHAI) takes the responsibility of revenue collection through tolls and maintains the project. The developer has no role in the maintenance of the project.

→ Question-97

Answer: b

Explanation:

Statement 1 is incorrect: The Council of Scientific & Industrial Research (CSIR) has conducted Whole Genome Sequencing of 1,008 Indians from different populations across the country. It was implemented by the CSIR-Institute of Genomics and Integrative Biology (IGIB), Delhi and CSIR-Centre for Cellular and Molecular Biology (CCMB), Hyderabad. There is no connection with WHO in this project.

Statement 2 is correct: The outcomes of the IndiGen will have applications in a number of areas including predictive and preventive medicine with faster and efficient diagnosis of rare genetic diseases. The data will be important for building the knowhow, baseline data and indigenous capacity in the emerging area of Precision Medicine. The outcomes of the IndiGen will be utilized towards understanding the genetic diversity on a population scale, make available genetic variant frequencies for clinical applications and enable genetic epidemiology of diseases. The benefits of this initiative include epidemiology of genetic diseases to enable cost effective genetic tests, carrier screening applications for expectant couples, enabling efficient diagnosis of heritable cancers and pharmacogenetic tests to prevent adverse drug reactions.

Statement 3 is correct: The Council of Scientific & Industrial Research (CSIR) has conducted Whole Genome Sequencing of 1,008 Indians from different populations across the country. IndiGen programme aims to undertake whole genome sequencing of thousands of individuals representing diverse ethnic groups from India.

→ Question-98

Answer: b

Explanation:

  • A US$165 million loan agreement was signed between The Government of India, the state Government of Odisha and the World Bank to support smallholder farmers in strengthening the resilience of their production systems, diversifying and improving the marketing of their produce, to realize more income.
  • The project will be implemented in rural areas vulnerable to droughts and largely dependent on rain-fed agriculture.
  • The project will strengthen the resilience of smallholder farmers against adverse climate.
  • It will try to improve access to resilient seed varieties and production technologies, diversifying towards more climate-resilient crops, and improving access to better water management and irrigation services.
  • The project will support the rehabilitation of 532 water tanks; promote productivity improvements at the farm level; support farmers to reduce the current emphasis on food grains (especially paddy and wheat) and increase the share of high-value and more nutritious products like fruits and vegetables; and provide marketing support to farmers who are able to generate a marketable surplus.
  • The project will also support aquaculture in rehabilitated tanks, help farmers to access affordable and quality fingerlings, and disseminate improved aquaculture practices and post-harvest management.

→ Question-99

Answer: d

Explanation:

Statement 1 is correct: Rheumatic fever is endemic in India and remains one of the major causes of cardiovascular disease, accounting for nearly 25-45% of acquired heart disease. The most devastating effects are on children and young adults in their most productive years.

Statement 2 is correct: In October 2019, the government was planning to procure penicillin centrally for three years and give it to all children between 5-15 years who have a sore throat, at least once. The Government is trying to revive penicillin availability because it is the cheapest option for rheumatic fever treatment.

Statement 3 is correct: Rheumatic heart disease disproportionately affects girls and women. The risk of developing rheumatic heart disease is up to two times higher for females than males.

→ Question-100

Answer: b

Statement 1 is incorrect: ICDS is a scheme for children from 0-6 years as well as pregnant and lactating mothers. Anganwadis are day care centres for children and are set up under ICDS. They are honorary workers who are paid a monthly honoraria.

Statement 2 is correct: It offers six services supplementary nutrition, pre-school non formal education, nutrition and health education, immunization, health check-up and referral services.

Statement 3 is correct: Rajiv Gandhi Scheme for Empowerment of Adolescent Girls (RGSEAG) or SABLA is implemented under ICDS through Anganwadi Centres (AWCs). The scheme is aimed at self-development, empowerment, improve awareness on nutrition and health status of adolescent girls.

UPSC Mock Test Description IAS Mock Test for Prelims  – General Studies Paper 1
Number of Questions 100
Type of Questions Objective-type (MCQs)
Time Duration 2 hours
Answer Key + Explanation Yes
Score Calculation Calculate UPSC Prelims score

UPSC mock tests help the aspirants to acclimatize themselves to the IAS Exam Pattern and candidates can also assess their strengths and weaknesses.

UPSC Prelims 2022 Exam Date – 5th June 2022

Since the UPSC Prelims 2021 has already been conducted, it is time now to solve the IAS mock tests to ace the UPSC 2022 examination. For UPSC aspirants, BYJU’S experts have created IAS mock tests after careful analysis of previous years’ question papers and trends.

Candidates can also check the subject-wise weightage in UPSC Prelims at the linked article.

Note: For best results, candidates should take this IAS mock test in the stipulated time limit (with the help of the timer given) and go through the explanations of the answers carefully.

Check the list of some important issues that can come helpful in prelims examination of civil services:

UPSC Prelims 2022 Related Links:
IAS Exam UPSC Prelims CSAT NCERT Notes UPSC Mains Syllabus
UPSC Exam Pattern UPSC Syllabus UPSC Previous Year Question Papers UPSC Preparation
UPSC Answer Keys IAS Current Affairs UPSC Prelims UPSC Online

Frequently Asked Questions on IAS Mock Test

Q1

Q 1. Which is the best mock test series for UPSC preparation?

Ans. There are multiple options available for practice series, both online and offline. One of the best options though is the BYJU’S IAS Mock Test series that comprise questions based on the latest exam pattern and syllabus. These are absolutely free of cost and can be accessed by any aspirant.
Q2

Q 2. Can one solve the same IAS mock test multiple times?

Ans. Yes, candidates can solve every test paper multiple times while preparing for the civil services exam. It is recommended that the last month before the date of the examination must be spent on solving as many mock series as possible.
Q3

Q 3. What are the benefits of solving IAS mock tests?

Ans. Discussed below are the advantages of solving UPSC test series:

  • Better understanding of the exam pattern
  • Apprehend the important topics from the examination perspective
  • A mock experience of the atmosphere in the examination hall
  • Comprehend the standard of examination
  • Analyse individual strengths and weaknesses
  • Complete revision at the same time
Q4

Q 4. Does the BYJU’S IAS mock test series provide answers as well?

Ans. Yes, an answer key is also provided along with the practice series. Aspirants can attempt the paper and then check their scores by comparing reviewing the answer key available.
UPSC Previous Year Question Paper Topic-wise IAS Prelims Question and Answers
UPSC Prelims Question Paper 2021 Topic-wise Essay Questions From UPSC Mains

Online Quiz 2022

Comments

Leave a Comment

Your Mobile number and Email id will not be published.

*

*